LSAT Explanation PT 30, S4, Q24: Telephone companies are promoting "voice mail"

LSAT Question Stem

Which one of the following, if true, most calls into question the argument made by the companies promoting voice mail? 

Logical Reasoning Question Type

This is a Weaken question. 

Correct Answer

The correct answer to this question is D. 

LSAT Question Complete Explanation

First, let's analyze the argument in the passage. The telephone companies are promoting voice mail as an alternative to answering machines, arguing that voice mail will soon make answering machines obsolete because it is more convenient, flexible, and less expensive. The conclusion of the argument is that voice mail will make answering machines obsolete soon. The premises supporting this conclusion are that voice mail is more convenient, flexible, and less expensive than answering machines.

An "Evaluate" question for this argument could be: "Are there any benefits of answering machines that voice mail does not provide?"

Now let's discuss the question type and the answer choices. The question type is Weaken, which means we're looking for an answer choice that calls into question or weakens the conclusion that voice mail will make answering machines obsolete soon.

a) This answer choice actually strengthens the argument by providing another benefit of voice mail over answering machines, so it is incorrect.

b) The sales history of answering machines does not address the benefits of voice mail compared to answering machines, so it is irrelevant and does not weaken the argument.

c) The fact that customers can cancel voice mail service at any time does not necessarily mean they will. This answer choice is too ambiguous and does not address the benefits of voice mail compared to answering machines, so it does not weaken the argument.

d) This answer choice presents a benefit of answering machines that voice mail does not provide, which weakens the argument that voice mail will make answering machines obsolete. This is the correct answer.

e) This answer choice highlights a limitation of answering machines, but it does not provide a comparison between voice mail and answering machines. Therefore, it does not weaken the argument.

In summary, the correct answer is D because it presents a benefit of answering machines that voice mail does not provide, which weakens the argument that voice mail will make answering machines obsolete soon.

Previous
Previous

LSAT Explanation PT 31, S2, Q22: On the surface, Melville's Billy Budd

Next
Next

LSAT Explanation PT 30, S2, Q14: It is inaccurate to say that